2025 AMC 10A Problems/Problem 5
Consider the sequence of positive integers
What is the
th term in the sequence?
Video Solution
Solution 1
One possible way the sequence could've been constructed was by putting "mountains" going up from
, to
then going back down to
For example, the first few "mountains" look like this:
So, the
mountain has length
and has highest number
We want to add mountains until we get a total length as close as possible, but not exceeding,
Let the last mountain we sum be mountain
Hence,
so our max
is
In this
mountain, the max number is
so the
mountain has max number
Next,
so we're looking for the
number in the
mountain, which is
~Tacos_are_yummy_1
Solution 2 (nice equation)
Group the numbers by their hill pattern:
The maximums of each hill occur at terms
These terms correspond to maximums of
Let
be the maximum at term
. Since the sum of the first
odd numbers is
we have
So for example, if a =
then
telling us that the peak of the hill with maximum
occurs at the
th term.
Now, we know
so let
Then
so the
th term is
Then the
th term must be
Solution 3
Note that the first
is the first number from the left, the second
is the third number from the left, and the third
is the seventh number from the left. Since the second differences between the indices of the
’s are equal, there is a quadratic function for the position of the
. As the second differences are
, the leading coefficient of this function is
. Let the function be
. Then, we have
, and
. Solving, we get
and
. Therefore, the
th
from the left is the
th number from the left. Plugging in
gives
, so the
st number is a
.
Also, the sequence shows that the first instance of a positive integer
is always to the left of the
. This means that following the
number, the numbers go to or above
. Adding
to
means that the
term of the sequence is
~joiceeliu
~minor
edits by i_am_not_suk_at_math (saharshdevaraju 11:09, 7 November 2025 (EST)saharshdevaraju)
Solution 4 (Quick and Fast!🚀)
Note that after every round, the number of terms is the highest term squared. For example, after the threes, there are
terms. We also notice that
, so the
th term is
, which is answer choice
~iiiiiizh
~minor
edits by i_am_not_suk_at_math (saharshdevaraju 09:35, 7 November 2025 (EST)saharshdevaraju)
Solution 5 (Super Simple!!)
Upon inspecting the problem, we see that the
term is 1, and so are the
,
,
,
,
, and so on. We see that the pattern comes from the differences:
,
,
,
,
, and so on. From this we see that a
appears at each position that is a number of the form
. We know that
. Plugging
as
gives
, meaning that the
term is
. Therefore, the solution is
~vgarg
~major
edits by i_am_not_suk_at_math (saharshdevaraju 15:27, 7 November 2025 (EST)saharshdevaraju)
Solution 6 - Fastest and Easiest!
\(\large \textbf{Takes less than 3 minutes!}\)
Immediately by looking at the sequence, we observe that the digit in the place of each perfect square is the square root of that number. For example, the first term in the sequence is \(1\) (because \(\sqrt{1}=1\)), the \(4^{th}\) term is \(2\) (because \(\sqrt{4}=2\)), the \(9^{th}\) term is \(3\) (because \(\sqrt{9}=3\)), the \(16^{th}\) term is \(4\) (because \(\sqrt{16}=4\)), and so on. And since \(2025\) is a perfect square, it makes it easy for us because \(\sqrt{2025}=45\), which makes the answer choice \(\boxed{\textbf{(E) }45}.\)
To make it easier to understand my way of solving it, here is a table.
Let \(n =\) the position of the said number in the sequence
Let \(\sqrt{n} =\) the number located in that position itself
\begin{array}{c|c|c} n & \sqrt{n} \\[6pt] \hline 1 & 1 \\[6pt] 4 & 2 \\[6pt] 9 & 3 \\[6pt] 16 & 4 \\[6pt] 25 & 5 \\[6pt] 36 & 6 \\[6pt] \dots & \dots \\[6pt] 2025 & \boxed{\textbf{(E) }45} \end{array}
\(\textbf{Note}\): I am just saying that the square root of the position (if the position is a perfect square) is the number that appears in the said position in the above sequence. I am not saying that the root itself is the first time it appears in the said sequence. To elaborate, the number \(6\) appears on the \(26^{th}\) number, as \(\textit{well}\) as the \(36^{th}\) number as expected (because \(\sqrt{36} = 6\)).
~i_am_not_suk_at_math (saharshdevaraju 21:14, 6 November 2025 (EST)saharshdevaraju)
Solution 7 - Pattern Recognition
After looking at the problem, it is evident that a pattern exists for the first occurence of a number in the sequence. Knowing this, we can write the places that each number appears at:
\begin{align}
&1: 1 \\
&2: 2 \\
&3: 5 \\
&4: 10 \\
&5: 17
\end{align}
If we write down the difference between the each numbers position in sequence, we get
. We notice how these numbers are similar to how the square numbers increase (square numbers:
). We also notice how the position of the first instance of each number
is just
. Since we're trying to find the
number and
, we can plug in
for
. As a result, we get that the first instance of the number
appears at
or
. Knowing that the number before the first instance of a number
is
, we get the answer choice
~Kasoisanti
~minor edits by i_am_not_suk_at_math (saharshdevaraju 09:34, 7 November 2025 (EST)saharshdevaraju)
~minor
edits by i_am_not_suk_at_math (saharshdevaraju 15:22, 7 November 2025 (EST)saharshdevaraju)
Solution 8 - Simple solution for dummies
We can list out all of the 2025 terms. If each term takes about 1 second, then it will take about 34 minutes in total, which is good enough if you are aiming for a 42 on the AMC 10. After listing all these terms, we can get the answer choice
~dingpanda
~this is actually really funny -i_am_not_suk_at_math
Solution 9: Patterns
We list the positions of the
in the sequence and get that they appear at
We see that the
in the sequence, its location is at the
position. So, we can find that the
closest to
happens where
due to our knowledge that
Since
we can start counting from
getting that the 2025th term in the sequence is
~ gogogo2022
Solution 10: Sum of Series
We have the following series given by the question:
Noting the differences between two consecutive numbers
gives us the following:
Notice that the problem now becomes finding the sum of the first 2025 numbers, instead of the 2025th number. We also notice that every
gets canceled eventually, and from inspection, the last
in a consecutive string of
s is always corresponding to the number at position
For example, the 9th number is the last
in the
consecutive numbers, and since the numbers before these consecutive
s cancel out, the sum (
) is the 9th number in the original series. So our answer must be
~ Cheetahboy93
Chinese Video Solution
https://www.bilibili.com/video/BV1gV2uBbEJe/
~metrixgo
Video Solution by SpreadTheMathLove
https://www.youtube.com/watch?v=dAeyV60Hu5c
Video Solution
~MK
Video Solution (Fast and easy)
https://youtu.be/ZQhAdIs2FIg?si=mEgnYvdQ2sMrCx1P ~ Pi Academy
Video Solution by Daily Dose of Math
~Thesmartgreekmathdude
See Also
| 2025 AMC 10A (Problems • Answer Key • Resources) | ||
| Preceded by Problem 4 |
Followed by Problem 6 | |
| 1 • 2 • 3 • 4 • 5 • 6 • 7 • 8 • 9 • 10 • 11 • 12 • 13 • 14 • 15 • 16 • 17 • 18 • 19 • 20 • 21 • 22 • 23 • 24 • 25 | ||
| All AMC 10 Problems and Solutions | ||
These problems are copyrighted © by the Mathematical Association of America.